subject
Mathematics, 02.02.2020 09:44 milagrosee12

Acell phone plan has a monthly cost that is shown in the table below. what is the correct statement regarding the average rate of change during the 40 minute time of talk?

total minutes of talk time:
0
10
20
30
40
monthly cost of cell phone
9.95
10.65
11.35
12.05
12.75

ansver
Answers: 1

Another question on Mathematics

question
Mathematics, 22.06.2019 00:00
Tony is charged $ 50 and additional $0.15 per miles for renting a car. a) represent the cost of renting a car with an equation,and the determine the cost if he drove it70 miles.b) what would be the cost of a car rental if the car was driven 250 miles? show work
Answers: 1
question
Mathematics, 22.06.2019 00:20
Nthe opening scene of the movie raiders of the lost ark, indiana jones replaces a golden statue with a bag of sand. the platform on which the statue is placed is designed to detect the mass of the statue. if the bag of sand has a different mass than the statue, then a mechanism triggers and destruction ensues. the density of gold is about 20\text{ g/cm}^320 g/cm 3 20, space, g, slash, c, m, start superscript, 3, end superscript. the density of sand can be estimated at 2.5\text{ g/cm}^32.5 g/cm 3 2, point, 5, space, g, slash, c, m, start superscript, 3, end superscript. the gold statue can be modeled as a cylinder with height 25\text{ cm}25 cm25, space, c, m and radius 8\text{ cm}8 cm8, space, c, m. in terms of \piπpi, what is the mass of sand indiana jones needs to avoid triggering the mechanism?
Answers: 2
question
Mathematics, 22.06.2019 06:30
1.5x + 0.2y = 2.68 1.6x + 0.3y = 2.98 0 the lines are parallel. the lines overlap at all points. the lines intersect at (1.6,1.4). the lines intersect at (3.1,0.5). which statement describes the graph of the system of equations below
Answers: 1
question
Mathematics, 22.06.2019 06:30
Use the rules of exponents to simplify the expressions. match the expression with its equivalent value. 1. 2-1 ∙ 2-4 2. -32 3. ∙ 32 4. -
Answers: 3
You know the right answer?
Acell phone plan has a monthly cost that is shown in the table below. what is the correct statement...
Questions
question
Mathematics, 14.12.2020 07:30
question
Mathematics, 14.12.2020 07:30
question
SAT, 14.12.2020 07:30
question
History, 14.12.2020 07:30
question
Spanish, 14.12.2020 07:30
question
Mathematics, 14.12.2020 07:30
question
Mathematics, 14.12.2020 07:30
question
Mathematics, 14.12.2020 07:30
Questions on the website: 13722359